[obm-l] mais de probabilidade

2004-10-13 Por tôpico Murilo Neves
Olá
Continuo aqui estudando e surgiram mais duas dúvidas. Se alguém puder me 
ajudar... Vamos lá:

1)Sabendo-se que E(X)=2 e que E(X^2)=4, calcule o menor valor possível para 
P(-10X14).
Meu gabarito dá que a resposta é aproximadamente 0.9. Tentei usar Chebyshev, 
mas como Var(X)=E^2(X) - E(X^2)=0, chegaria a resposta 1. Tentei usar 
Markov, mas também não consegui.

2) Três cruzadores A, B e C, atacam um navio inimigo. Na primeira leva de 
tiros, o cruzador A tem 0.2 de probabilidade de atingir o alvo; enquanto os 
cruzadores B e C têm, respectivamente, 0.4 e 0.1 de probabilidade. Se o 
navio inimigo receber tiros de apenas um dos navios, ele tem prob 0.05 de 
afundar imediatamente; se receber de dois, a prob aumenta para 0.2 e se 
receber de três, a prob é de 0.4. Qual a prob. do navio inimigo afundar na 
primeira leva de tiros?

Obrigado
_
MSN Hotmail, o maior webmail do Brasil.  http://www.hotmail.com
=
Instruções para entrar na lista, sair da lista e usar a lista em
http://www.mat.puc-rio.br/~nicolau/olimp/obm-l.html
=


[obm-l] Duvidas

2004-10-13 Por tôpico aryqueirozq


Se a função linear f , dada por f(x) = ax + b , satifaz 
a condição f ( 5x +2 ) = 5 f(x) +2 , pode-se afirmar 
então que:

a) a= 2b
b) a= b +2
c) a= 2( b+ 2)
d) a= 2(b+1)
e) a=2b+1


Agradeço desde de já.
 
__
Acabe com aquelas janelinhas que pulam na sua tela.
AntiPop-up UOL - É grátis!
http://antipopup.uol.com.br/



=
Instruções para entrar na lista, sair da lista e usar a lista em
http://www.mat.puc-rio.br/~nicolau/olimp/obm-l.html
=


Re: [obm-l] [obm-1] Probabilidade

2004-10-13 Por tôpico Leonardo Paulo Maia
Edward, esse problema é um clássico, chama-se problema da agulha de Buffon e é
tradicionalmente resolvido usando probabilidade geométrica (calculam-se
probabilidades mediante razões entre áreas, volumes, ...). O texto de mais
fácil acesso que trata desse problema é o livro de Cálculo c/ GA, do Simmons,
vol. 1, pág. 428, mas o número 20 da RPM (Revista do Prof. de Matemática), pág.
16, também mostra a solução, em um artigo geral sobre prob. geom. . Procure uma
dessas refs., é difícil dar uma explicação clara em pouco espaço. Se você não
tiver acesso aos dois, escreva novamente e eu preparo uma solução cuidadosa.
Leo

Quoting Edward Elric [EMAIL PROTECTED]:

 Eis um problema de probabilidade que me parece de um nivel consideravel:
 
 Considere uma área plana, dividida em faixas de larguras iguais, a, por 
 retas paralelas. Lance sobre a regiao, ao acaso, uma agulha de comprimento 
 2r, com 2ra. Qual a probabilidade de que a agulha corte umas das paralelas?
 
 Eu nao consegui, seria bom uma ajuda :)
 
 _
 MSN Hotmail, o maior webmail do Brasil.  http://www.hotmail.com
 
 =
 Instruções para entrar na lista, sair da lista e usar a lista em
 http://www.mat.puc-rio.br/~nicolau/olimp/obm-l.html
 =
 


=
Instruções para entrar na lista, sair da lista e usar a lista em
http://www.mat.puc-rio.br/~nicolau/olimp/obm-l.html
=


Re: [obm-l] [obm-1] Probabilidade

2004-10-13 Por tôpico Leonardo Paulo Maia
Edward, esse problema é um clássico, chama-se problema da agulha de Buffon e é
tradicionalmente resolvido usando probabilidade geométrica (calculam-se
probabilidades mediante razões entre áreas, volumes, ...). O texto de mais
fácil acesso que trata desse problema é o livro de Cálculo c/ GA, do Simmons,
vol. 1, pág. 428, mas o número 20 da RPM (Revista do Prof. de Matemática), pág.
16, também mostra a solução, em um artigo geral sobre prob. geom. . Procure uma
dessas refs., é difícil dar uma explicação clara em pouco espaço. Se você não
tiver acesso aos dois, escreva novamente e eu preparo uma solução cuidadosa.
Leo

Quoting Edward Elric [EMAIL PROTECTED]:

 Eis um problema de probabilidade que me parece de um nivel consideravel:
 
 Considere uma área plana, dividida em faixas de larguras iguais, a, por 
 retas paralelas. Lance sobre a regiao, ao acaso, uma agulha de comprimento 
 2r, com 2ra. Qual a probabilidade de que a agulha corte umas das paralelas?
 
 Eu nao consegui, seria bom uma ajuda :)
 
 _
 MSN Hotmail, o maior webmail do Brasil.  http://www.hotmail.com
 
 =
 Instruções para entrar na lista, sair da lista e usar a lista em
 http://www.mat.puc-rio.br/~nicolau/olimp/obm-l.html
 =
 


=
Instruções para entrar na lista, sair da lista e usar a lista em
http://www.mat.puc-rio.br/~nicolau/olimp/obm-l.html
=


Re: [obm-l] [obm-1] Probabilidade

2004-10-13 Por tôpico Leonardo Paulo Maia
Edward, esse problema é um clássico, chama-se problema da agulha de Buffon e é
tradicionalmente resolvido usando probabilidade geométrica (calculam-se
probabilidades mediante razões entre áreas, volumes, ...). O texto de mais
fácil acesso que trata desse problema é o livro de Cálculo c/ GA, do Simmons,
vol. 1, pág. 428, mas o número 20 da RPM (Revista do Prof. de Matemática), pág.
16, também mostra a solução, em um artigo geral sobre prob. geom. . Procure uma
dessas refs., é difícil dar uma explicação clara em pouco espaço. Se você não
tiver acesso aos dois, escreva novamente e eu preparo uma solução cuidadosa.
Leo


Quoting Edward Elric [EMAIL PROTECTED]:

 Eis um problema de probabilidade que me parece de um nivel consideravel:
 
 Considere uma área plana, dividida em faixas de larguras iguais, a, por 
 retas paralelas. Lance sobre a regiao, ao acaso, uma agulha de comprimento 
 2r, com 2ra. Qual a probabilidade de que a agulha corte umas das paralelas?
 
 Eu nao consegui, seria bom uma ajuda :)
 
 _
 MSN Hotmail, o maior webmail do Brasil.  http://www.hotmail.com
 
 =
 Instruções para entrar na lista, sair da lista e usar a lista em
 http://www.mat.puc-rio.br/~nicolau/olimp/obm-l.html
 =
 


=
Instruções para entrar na lista, sair da lista e usar a lista em
http://www.mat.puc-rio.br/~nicolau/olimp/obm-l.html
=


Re: [obm-l] [obm-1] Probabilidade

2004-10-13 Por tôpico Felipe Amaral
Acho que nao entendi o problema direito pois com a resposta do Marcio:

r = a   =   p  1

ou seja sempre cortaria... Mas a agulha ainda pode cair de lado certo,
entao a probabilidade deveria ser menor do que 1?

Grato, Amaral
=
Instruções para entrar na lista, sair da lista e usar a lista em
http://www.mat.puc-rio.br/~nicolau/olimp/obm-l.html
=


Re: [obm-l] Duvidas

2004-10-13 Por tôpico douglas . rodrigues




Qdo x = 0, f(0) = b.

Além disso, f(5.0 + 2) = 5.f(0) + 2 = 5.b + 2. Como f(2) = a.2 + b, temos
que:

5.b + 2 = 2.a + b = a = 2.b + 1

Acho que é isso.

Abraço,

Douglas Felipe Rodrigues da Silva



   

  aryqueirozq

  [EMAIL PROTECTED]To:   obm-l [EMAIL PROTECTED]  
  
  com.br  cc: 

  Sent by: Subject:  [obm-l] Duvidas   

  [EMAIL PROTECTED]

  uc-rio.br

   

   

   

   

  13/10/04 08:37   

  Please respond to

  obm-l

   

   







Se a função linear f , dada por f(x) = ax + b , satifaz
a condição f ( 5x +2 ) = 5 f(x) +2 , pode-se afirmar
então que:

a) a= 2b
b) a= b +2
c) a= 2( b+ 2)
d) a= 2(b+1)
e) a=2b+1


Agradeço desde de já.

__
Acabe com aquelas janelinhas que pulam na sua tela.
AntiPop-up UOL - É grátis!
http://antipopup.uol.com.br/



=
Instruções para entrar na lista, sair da lista e usar a lista em
http://www.mat.puc-rio.br/~nicolau/olimp/obm-l.html
=




=
Instruções para entrar na lista, sair da lista e usar a lista em
http://www.mat.puc-rio.br/~nicolau/olimp/obm-l.html
=


Re: [obm-l] Duvidas

2004-10-13 Por tôpico Leonardo Paulo Maia
f(5x+2) = a(5x+2)+b

5f(x)+2 = 5ax+5b+2

Então f(5x+2) = 5f(x)+2 - a(5x+2)+b = 5ax+5b+2
- 2a+b = 5b+2
- a = 2b+1

Quoting aryqueirozq [EMAIL PROTECTED]:

 
 
 Se a função linear f , dada por f(x) = ax + b , satifaz 
 a condição f ( 5x +2 ) = 5 f(x) +2 , pode-se afirmar 
 então que:
 
 a) a= 2b
 b) a= b +2
 c) a= 2( b+ 2)
 d) a= 2(b+1)
 e) a=2b+1
 
 
 Agradeço desde de já.
  
 __
 Acabe com aquelas janelinhas que pulam na sua tela.
 AntiPop-up UOL - É grátis!
 http://antipopup.uol.com.br/
 
 
 
 =
 Instruções para entrar na lista, sair da lista e usar a lista em
 http://www.mat.puc-rio.br/~nicolau/olimp/obm-l.html
 =
 


=
Instruções para entrar na lista, sair da lista e usar a lista em
http://www.mat.puc-rio.br/~nicolau/olimp/obm-l.html
=


Re: [obm-l] [obm-1] Probabilidade

2004-10-13 Por tôpico Celso Souza
 Este é o conhecido problema de Buffon. Em aulas eu já utilizei este problema para calcularmos experimentalmente o valor de Pi.

 Você encontra referências em :

http://www.mste.uiuc.edu/reese/buffon/buffon.html

http://www.cut-the-knot.org/fta/Buffon/buffon9.shtml

http://www-history.mcs.st-and.ac.uk/~history/Mathematicians/Buffon.html

 Espero ter ajudado.

Fraternalmente,

Celso Faria de Souza
		Yahoo! Acesso Grátis - Internet rápida e grátis. Instale o discador agora!

Re: [obm-l] probabilidade e estatística

2004-10-13 Por tôpico Leonardo Paulo Maia
1-) falso: o correto é g(m)=n{[1-F(m)]^(n-1)]}*f(m)

Lembrando que f(m)=F'(m) e, analogamente, defino G(m) = P(M=m) de forma que
g(m)=G'(m).

G(m) = 1-P(Mm) = 1-[P(Xim)]^n = 1-[1-F(m)]^n

Derivando,

g(m) = -n {[1-F(m)]^(n-1)} {-F'(m)} = n{[1-F(m)]^(n-1)]}*f(m)

2-) passo!

extra-) Y=F(X) é uniformemente distribuída em [0,1] e seu raciocínio está
correto.

Leo

Quoting Murilo Neves [EMAIL PROTECTED]:

 Olá
 
   Estou com dúvida em dois exercícios do tipo Verdadeiro ou Falso ( 
 justificando):
 
 1)Sejam: X uma v.a. contínua com fdp f e fda F e X1,...,Xn uma amostra 
 aleátória de X. Se M é o valor mínimo da amostra, então a fdp de M será dada
 
 por g(m)=n{[F(m)]^(n-1)]}*f(m)
 
 2) Seja o modelo de regressão yt=B1+B2*yt + et, onde xt é não estocástica e 
 et satisfaz as hipóteses usuais do modelo de regressão. O R^2 dessa 
 regressão será extamente igual a 1 se, e só se, o valor do estimador de MQO 
 de D2 da equação xt = D1 + D2*yt + ut for exatamente igual ao inverso do 
 valor do estimador de MQO de B2.
 
 Tinha uma cujo enunciado era:
 Seja X um va contínua com fdp f e fda F. Defina Y como outra va tal que 
 Y=F(X). Então F será uniformemente distribuida sobre [0,1]
 
 Minha resposta: Como X é contínua e F é não decrescente, temos que existe a 
 inversa de F, F^-1. Seja Fy a fda de Y. Daí 
 Fy(a)=P(Ya)=P(F(X)a)=P(XF^-1(a))=F[F^-1(a)]=a.Logo Fy(y)=y e, assim, 
 fy(y)=1. Resposta: Verdadeiro. Isto está correto?
 
 Obrigado por qualquer ajuda.
 
 Murilo
 
 _
 MSN Hotmail, o maior webmail do Brasil.  http://www.hotmail.com
 
 =
 Instruções para entrar na lista, sair da lista e usar a lista em
 http://www.mat.puc-rio.br/~nicolau/olimp/obm-l.html
 =
 


=
Instruções para entrar na lista, sair da lista e usar a lista em
http://www.mat.puc-rio.br/~nicolau/olimp/obm-l.html
=


Re: [obm-l] mais de probabilidade

2004-10-13 Por tôpico Leonardo Paulo Maia
1-) passo!

2-) Tente construir P(afundar) = (soma com k de 0 a 3) P(afundar | k ataques) .
P(k ataques)

P(afundar | k ataques) foi dado diretamente no enunciado (é claro que P(afundar
| 0 ataques)=0 ) e você pode combinar as probs. dos cruzadores atingirem o alvo
p/ obter P(k ataques).

Leo

Quoting Murilo Neves [EMAIL PROTECTED]:

 Olá
 
 Continuo aqui estudando e surgiram mais duas dúvidas. Se alguém puder me 
 ajudar... Vamos lá:
 
 1)Sabendo-se que E(X)=2 e que E(X^2)=4, calcule o menor valor possível para 
 P(-10X14).
 Meu gabarito dá que a resposta é aproximadamente 0.9. Tentei usar Chebyshev,
 
 mas como Var(X)=E^2(X) - E(X^2)=0, chegaria a resposta 1. Tentei usar 
 Markov, mas também não consegui.
 
 2) Três cruzadores A, B e C, atacam um navio inimigo. Na primeira leva de 
 tiros, o cruzador A tem 0.2 de probabilidade de atingir o alvo; enquanto os 
 cruzadores B e C têm, respectivamente, 0.4 e 0.1 de probabilidade. Se o 
 navio inimigo receber tiros de apenas um dos navios, ele tem prob 0.05 de 
 afundar imediatamente; se receber de dois, a prob aumenta para 0.2 e se 
 receber de três, a prob é de 0.4. Qual a prob. do navio inimigo afundar na 
 primeira leva de tiros?
 
 Obrigado
 
 _
 MSN Hotmail, o maior webmail do Brasil.  http://www.hotmail.com
 
 =
 Instruções para entrar na lista, sair da lista e usar a lista em
 http://www.mat.puc-rio.br/~nicolau/olimp/obm-l.html
 =
 


=
Instruções para entrar na lista, sair da lista e usar a lista em
http://www.mat.puc-rio.br/~nicolau/olimp/obm-l.html
=


Re: [obm-l] [obm-1] Probabilidade

2004-10-13 Por tôpico Marcio Cohen
Oi Felipe, acho que voce entendeu o problema sim, mas esqueceu de um
dado dele. Ele diz que 2r  a no enunciado!
O caso em que 2r = a (ou seja, a agulha eh grande o suficiente para
cortar duas vezes) eh sensivelmente mais complicado e tem como resposta uma
expressao bem mais feia, envolvendo raizes e funcoes trigonometricas
inversas.
Abraco,
Marcio

PS: Eu usei que 2r  a quando disse que o numero esperado de cruzamentos
E(x) era igual a probabilidade procurada (se a agulha for maior que a, entao
E(x) = p1 + 2p2 + 3p3 +..., onde p_i eh a probabilidade de haver i
cruzamentos. no nosso caso, p2=p3=...=0 e portanto E(x) = p1).

- Original Message - 
From: Felipe Amaral [EMAIL PROTECTED]
To: [EMAIL PROTECTED]
Sent: Wednesday, October 13, 2004 8:56 AM
Subject: Re: [obm-l] [obm-1] Probabilidade


 Acho que nao entendi o problema direito pois com a resposta do Marcio:

 r = a   =   p  1

 ou seja sempre cortaria... Mas a agulha ainda pode cair de lado certo,
 entao a probabilidade deveria ser menor do que 1?

 Grato, Amaral
 =
 Instruções para entrar na lista, sair da lista e usar a lista em
 http://www.mat.puc-rio.br/~nicolau/olimp/obm-l.html
 =




=
Instruções para entrar na lista, sair da lista e usar a lista em
http://www.mat.puc-rio.br/~nicolau/olimp/obm-l.html
=


Re: [obm-l] Problema Legal

2004-10-13 Por tôpico Claudio Buffara
on 12.10.04 18:09, benedito at [EMAIL PROTECTED] wrote:

 Abaixo, segue um problema legal:
 
 
 
 Problema
 
 Num corredor, existem  100  armários  em fila, numeradas de  1  até  100. Um
 pintor vem e pinta todas os armários de vermelho. Em seguida, vem um segundo
 pintor e pinta de azul os armários de três em três, começando do armário
 número 3. A seguir, vem um terceiro pintor e pinta de vermelho os armários
 de cinco em cinco, começando no armário de número 5 (ele pinta de vermelho,
 mesmo que o armário já seja vermelho). Em seguida, vem um quarto pintor e
 pinta de azul os armários de sete em sete, começando no armário  7. A
 seguir, vem um quinto pintor, e assim por diante, alternando a pintura
 vermelha, azul, até o pintor de número 50.
 
 No final, quantos armários são vermelhos?

 
As pinturas vermelhas sao multiplas de 1, 5, 9, 13, ..., 97 (4k+1)
As pinturas azuis sao multiplas de 3, 7, 11, 15, ..., 99 (4k+3)

A cor final de um dado armario eh a cor da ultima pintura que ele recebe e
esta cor corresponde justamente ao maior divisor impar do numero do armario.
Assim, o numero de armarios azuis eh igual ao numero de inteiros entre 1 e
100 (inclusive) cujo maior divisor impar eh da forma 4k+3.

Estes numeros sao:
3, 7, 11, 15, ..., 95, 99   (25);
6, 14, 22, 30, ..., 86, 94   (25+12=37);
12, 28, 44, 60, 76, 92   (37+6=43);
24, 56, 88   (43+3=46);
48   (46+1=47)

Logo, os armarios vermelhos sao em numero de 100 - 47 = 53.

[]s,
Claudio.



=
Instruções para entrar na lista, sair da lista e usar a lista em
http://www.mat.puc-rio.br/~nicolau/olimp/obm-l.html
=


Re: [obm-l] Dentro- da- Lei... Livros Gratuitos

2004-10-13 Por tôpico Valdery Sousa
Olá Rhilbert!

Ultimamente andei analisando os sites de livros na Internet que
vc envio e achei-os muito interessantes.
Vc tem outros? (Especialmente de Cálculo1...)
Sabe, concordo com vc em termos de livros grátis.
Aqui no Brasil os livros são muito caros e boa parte da grana do livro
não vai para o autor mas para a maldita editora.
Se vc quiser tenho uns sites de livros grátis e um grupo de discussões 
para o qual posso lhe convidar.


Cordiamente,
Valdery Sousa


_Rhilbert Rivera [EMAIL PROTECTED] wrote:
Não é preciso ser fora-da-lei para se conseguir muitos livros, apostilas e monografias gratuitas de Matemática na internet. Aí vão alguns endereços e... divirtam-sehttp://www.elprisma.com/apuntes/apuntes.asp?categoria=704http://www.numbertheory.org/ntw/lecture_notes.htmlhttp://www.math.miami.edu/~ec/book/http://www.maths.nott.ac.uk/personal/jec/courses/G13NUM/#linkhttp://f2.org/links/books.htmlTem mais outros, vou procurar com mais paciência e qualquer dia coloco na lista(^_^)_MSN Messenger: converse com os seus amigos online. http://messenger.msn.com.br=Instruções para entrar na lista, sair da lista e usar a lista
 emhttp://www.mat.puc-rio.br/~nicolau/olimp/obm-l.html=__Do You Yahoo!?Tired of spam?  Yahoo! Mail has the best spam protection around http://mail.yahoo.com 

[obm-l] Geom. Plana

2004-10-13 Por tôpico aryqueirozq
01. Um círculo é inscrito em um trapezóide ABCD.Tome 
K,L,M,N como os pontos de intersecço~es do círculo com 
as diagonais AC e BD respectivamente ( K entre A e L ,  
e M entre B e N ) . Sendo AK*LC = 16 e BM * ND = 9/4 , 
ache o raio do círculo.



02.Suponha que n( r ) denota o número de pontos com 
coordenadas inteiras em um círculo de raio r  1. Prove 
que n ( r )  6*raiz cúbica de pi*r^2 
  Agradeço.
 
__
Acabe com aquelas janelinhas que pulam na sua tela.
AntiPop-up UOL - É grátis!
http://antipopup.uol.com.br/



=
Instruções para entrar na lista, sair da lista e usar a lista em
http://www.mat.puc-rio.br/~nicolau/olimp/obm-l.html
=


Re: [obm-l] raiz(2+raiz(2+raiz(....

2004-10-13 Por tôpico Claudio Buffara
Soh pra completar o que o Artur disse: a sua solucao alternativa estah
correta, mas da mesma forma que antes, estah incompleta, faltando provar que
a sequencia que origina os radicais encaixados converge (o que o Artur chama
de comprovacao matematica que leva aos resultados desejados)

Agora, pode ser que a banca do vestibular - mesmo um puxado como o do IME -
queira apenas a resposta - o valor do limite - e considere como dada a
convergencia da sequencia correspondente. A questao passa, assim, a medir a
engenhosidade do candidato em manipulacoes algebricas, mas nao o seu
conhecimento dos fundamentos de analise matematica.

Acredito que, numa prova de um curso serio de analise, a sua solucao
ganharia apenas credito parcial.

*

Sobre o problema que voce mencionou, da equacao x^x^x^... = raiz(2), acho
que dah pra provar que existe um real a tal que a sequencia (x(n)), definida
por:
x(1) = b;
x(n+1) = b^x(n)  (n = 1)
converge se e somente se b = a  (pode ser que seja b  a).

Isso eh razoavel se voce considerar que a sequencia converge para 1 se b = 1
e diverge se b = 2. Logo, a deve pertencer a (1,2).
Se nao me engano, a = e^(1/e) ~ 1,444667... mas nao tenho 100% de certeza...


[]s,
Claudio.

on 11.10.04 17:17, Artur Costa Steiner at [EMAIL PROTECTED] wrote:

 Acho que uma resposta geral aa sua pergunta eh dificil. No caso da sequencia
 que ciculou na lista, era, de fato, essencial provar que a mesma era
 convergente antes de de resolvermos a equacao f(x) = x. Mas isto nao eh um
 conceito superior, eh algo basico para que estuda sequencias.
 O que se costuma chamar de artificio eh um processo, envolvendo manipulacoes
 algebricas, que permite resolver algum problema de forma rapida. O nome
 artificio vem de artificial, o que me parece uma denominacao infeliz. Como
 podemos definir o que eh natural e o que eh artificial? O importante eh que
 qualquer processo tem que ser matematicamente consistente e soh pode ser
 aplicado se houver comprovacao matematica de que leva aos resultados
 desejados. O problema eh que algumas vezes se aplicam estes processos
 chamados de artificios a situacoes e que eles nao soa validos.
 Quanto a se eh possivel resolver um problema sem recorrer a a conceitos de
 nivel superior, temos antes de mais nada que estabelecer o que eh conceito
 de nivel superior. Eh dificil dar uma resposta geral. As vezes eh possivel,
 dependendo do que se considera como superior. Por exemplo, eh possivel
 demonstrar recorrendo apenas aa Algebra, sem calculo diferencial, que
 trinomios do segundo grau tem um unico maximo ou um minimo, assim como eh
 possivel soh com a Algebra determinar para que valor ocorre este maximo ou
 minimo. Eh bem mais complicado do que com o calculo - com o qual a solucao
 eh imediata -, mas eh possivel.
 Mas soh com a Agebra me parece dificil mostrar que f(x) = x*e^x tem um
 minimo em x=-1.  
 Artur
 
 - Mensagem Original 
 De: [EMAIL PROTECTED]
 Para: obm-l [EMAIL PROTECTED]
 Assunto: [obm-l] raiz(2+raiz(2+raiz(
 Data: 08/10/04 19:29
 
 
 Para Claudio, e os amigos da lista
 Há pouco tempo enviei a solução abaixo e vc disse que
 soh estaria correta se soubesse que a serie realmente
 convergia. Corcordei plenamente, mas por outro lado
 fiquei pensando, pois esta questão e outras parecidas
 com ela (tipo x^x^x... = sqrt2) são questões que jah
 cairam no vestibular do ime, e a resolução apresentada
 foi justamente essa por artifíco. Pergunta existiria
 uma maneira de resolvê-las sem usar conceitos de nivel
 superior? 
 []s
 
 Outra soluçao:
 x = sqrt(2+sqrt(2+sqrt(2+sqrt(2+... (*)
 Seja u = sqrt(2+sqrt(2+sqrt(2+sqrt(2+...
 Note que (*) pode ser reescrita como:
 x = sqrt (u +2) e portanto:
 x = sqrt (x + 2)
 Elevando ambos os membros ao quadrado e resolv. a eq.
 do segundo grau , obtemos as raizes 2 e -1.
 Portanto a soluçao eh dois.
 []s
 
 
 
 -- Início da mensagem original ---
 
 De: [EMAIL PROTECTED]
 Para: quot;obm-lquot; [EMAIL PROTECTED]
 Cc: 
 Data: Wed, 6 Oct 2004 18:18:15 -0300
 Assunto: [obm-l] raiz(2+raiz(2+raiz(
 
 Seja (x(n)) a sequência definida por:
 x(1) = raiz(2)
 x(n+1) = raiz(2 + x(n)), para n = 1.
 
 1. (x(n)) é limitada:
 Basta provar que x(n)  2, para todo n.
 Para n = 1 é óbvio.
 Supondo que x(n-1)  2, teremos que x(n) = raiz(2 + x
 (n-1))  raiz(2 + 2) = 2 e acabou.
 
 2. (x(n)) é monótona crescente:
 Obviamente os x(n) são todos positivos.
 Assim, basta mostrar que x(n+1)^2  x(n)^2.
 Mas x(n+1)^2 - x(n)^2 = 2 + x(n) - x(n)^2  0 para 0
  x(n)  2.
 
 (1) e (2) implicam que (x(n)) converge. Seja x = lim x
 (n).
 
 Então, x^2 = 2 + x == x^2 - x - 2 = 0 == x = 2 ou x
 = -1.
 A raiz negativa deve ser descartada pois cada x(n) é
 positivo.
 
 Assim, só pode ser lim x(n) = 2.
 
 []s,
 Claudio.
 
 De:[EMAIL PROTECTED]
 
 Para:obm-l [EMAIL PROTECTED]
 
 Cópia:
 
 Data:Wed, 6 Oct 2004 16:59:33 -0300
 
 Assunto:[obm-l] Re:[obm-l] Re: [obm-l] Exercício
 
 
 
 x = sqrt(2+sqrt(2+sqrt(2+sqrt(2+...
 
 x^2 = 2 + 

Re: [obm-l] elementos de ordem 2 em grupos abelianos

2004-10-13 Por tôpico Claudio Buffara
on 12.10.04 02:00, [EMAIL PROTECTED] at [EMAIL PROTECTED] wrote:

 Eu estava comendo mosca. Se G é um grupo abeliano no qual todo elemento
 salvo a unidade tem ordem 2, então G tem 2^n elementos. Esse resultado segue
 do teorema de Cauchy. Porém ainda não dá para assegurar que dado n qualquer
 seja possível formar um grupo com 2^n elementos nessas condições, embora até
 n=3 tenha dado certo.

Que tal (Z_2)^n = espaco vetorial das n-uplas ordenadas cujas componentes
sao elementos de Z_2, ou seja, 0 ou 1, com a operacao de soma componente a
componente e tal que 0+0 = 1+1 = 0 e 0+1 = 1+0 = 1?
  
A hipotese de G ser abeliano e o teorema de Cauchy tambem sao
desnecessarios. Todo grupo finito G em que os elementos distintos da
identidade tem ordem 2 eh necessariamente abeliano e tem ordem 2^n para
algum n.

Dados a e b em G, teremos a^2 = b^2 = e == a^2b^2 = aabb = e.
Tambem (ab)^2 = abab = e. Ou seja, aabb = abab e cancelando a na esquerda e
b na direita obtemos ab = ba, o que prova que G eh abeliano.

Agora, tomemos um conjunto minimal de geradores do grupo G (uma base, por
assim dizer) x_1, x_2, ..., x_n. Cada elemento de G pode ser expresso de
forma unica da forma x_1^a_1*x^2^a_2*...*x_n^a_n, onde a_i eh um inteiro
nao-negativo para 1 = i = n.
Levando em conta que x_i^2 = e, vemos que podemos restringir cada a_i ao
conjunto {0,1}. Isso dah 2 alternativas para cada a_i, num total de 2^n
alternativas, uma para cada elemento de G. Logo |G| = 2^n.

[]s,
Claudio.

 [EMAIL PROTECTED] escreveu:
 
 É fácil mostrar que se G é um grupo abeliano, então ou não existe nenhum
 elemento de ordem 2 em G, ou então existe um número ímpar de elementos desse
 tipo; basta observar que juntamente com a unidade eles formam um subgrupo H
 de G e então usar Lagrange em cima de um subgrupo gerado por qualquer
 elemento (diferente da unidade) de H.
 
 No entanto, dado um x ímpar qualquer, nem sempre é possível formar um grupo
 abeliano que tenha x elementos de ordem 2; pelo menos eu desconfio disso.
 Fazendo algumas computações, consegui formar grupos abelianos com 1 e 3
 elementos de ordem 2, mas ao inserir um 4º elemento, acabei terminando com 7
 no total, ou seja, não consegui formar um grupo com 5 elementos de ordem 2.
 
 Enfim, alguém saberia dizer mais a esse respeito? Existe alguma regularidade
 na formação de grupos abelianos com elementos de ordem 2 (para facilitar a
 vida, considere um grupo onde todos os elementos diferentes da unidade têm
 ordem 2), isto é, os números possíveis de elementos desses grupos?
 
 []s,
 Daniel
 


=
Instruções para entrar na lista, sair da lista e usar a lista em
http://www.mat.puc-rio.br/~nicolau/olimp/obm-l.html
=


[obm-l] RE: [obm-l] livro disponibilizado on-line pelo própio autor

2004-10-13 Por tôpico Paulo Santa Rita
Ola Diogo e demais
colegas desta lista ... OBM-L,
Se, por um lado, todos nos que prezamos esta nossa LISTA DE DISCUSSAO DE 
PROBLEMAS DE MATEMATICA OLIMPICA, temos o dever de zelar por sua integridade 
e lisura, combatendo com certa energia praticas ilegais que eventualmente 
aparecem; por outro lado é claramente saudavel apoiar qualquer nobre atitude 
que aqui se manifeste...

Se o proprio autor disponibiliza um livro, a copia e a transmissao e legal e 
presta um grande favor as pessoas menos favorecidas quem o divulga, 
demonstrando portanto que alem de um intelecto educado tem tambem 
sentimentos educados.

Parabens por sua iniciativa !
um Abracao
Paulo Santa Rita
4,1119,131004
From: Diogo [EMAIL PROTECTED]
Reply-To: [EMAIL PROTECTED]
To: [EMAIL PROTECTED]
Subject: [obm-l] livro disponibilizado on-line pelo própio autor Date: Mon, 
11 Oct 2004 17:37:47 -0300

Este livro vem sendo usado no primeiro curso de matemática dos cursos de 
phd
em economia na upenn e nyu. Como já disse, o própio autor, Efe A. Ok, o
disponibilizou em sua página pessoal,  portanto, é legal.

http://homepages.nyu.edu/~eo1/books.html
_
MSN Messenger: converse com os seus amigos online.  
http://messenger.msn.com.br

=
Instruções para entrar na lista, sair da lista e usar a lista em
http://www.mat.puc-rio.br/~nicolau/olimp/obm-l.html
=


Re: [obm-l] Um de geometria do Claudio Buffara

2004-10-13 Por tôpico Claudio Buffara
Title: Re: [obm-l] Um de geometria do Claudio Buffara



A solucao que eu encontrei foi a seguinte:

Tome o ponto M tal que MBC eh semelhante a PQR e M e A se encontram no mesmo semi-plano determinado por BC. Nesse caso, teremos as igualdades de angulos:
BMD = BAD e CMD = CAD.
A ideia eh provar que M e A coincidem.

Para isso, construa os arcos capazes de BAD em relacao a BD e de CAD em relacao a DC.
Por serem arcos de circunferencia, eles se intersectam em exatamente dois pontos: A e D.
Como BMD = BAD, M pertence ao primeiro arco.
Como CMD = CAD, M pertence ao segundo arco.
Logo, M = D ou M = A.
Mas M eh claramente distinto de D, pois caso contrario, o triangulo MBC (e portanto o triangulo PQR) seria degenerado. 
Logo, soh pode ser M = A == CAD = CMD ~ PQR.

[]s,
Claudio.

on 10.10.04 19:20, Tércio Miranda at [EMAIL PROTECTED] wrote:

Problema
São dados os triângulos ABC e PQR, com medianas AD e PS , respectivamente. Valem as seguintes igualdades de
Ângulos, BAD=QPS e CAD=RPS. Prove que ABC e PQR são semelhantes.
 
Fixemos o triângulo ABC no seu plano.
Consideremos as semiretas AB e AC. Sobre elas marquemos os pontos L e M tal que AL=PQ e AM=QR. As hipóteses nos
dão as congruências dos triângulos PQR e ALM (LAL).
A reta suporte da mediana AD corta o segmento de reta LM num ponto K, o qual pelas hipóteses de igualdade de ângulos
BAD=QPS e CAD=RPS, é o ponto médio do segmento LM.
Agora, se, por absurdo, LM não for paralela a BC podemos conduzir por K uma paralela a BC que cortará AB e AC (semiretas)
nos pontos U e V, respectivamente. Daí UMVL seria um paralelogramo! Um contradição.
Então LM é paralela a BC e os triângulos ABC e ALM são semelhantes e temos o resultado.
 
Um abraço do colega 
Tércio Miranda
 







Re: [obm-l] Cordas no grafico de uma funcao

2004-10-13 Por tôpico Claudio Buffara
Eu achei uma solucao que usou sucessivamente o teorema do valor medio.
No entanto, uma dica que talvez seja util eh a seguinte: se as duas cordas
se bisectam, entao elas sao diagonais de um paralelogramo... pensando
melhor, no fim voce vai precisar do t.v.m de qualquer jeito.

[]s,
Claudio.

on 08.10.04 17:45, Artur Costa Steiner at [EMAIL PROTECTED] wrote:

 Fazendo uma analise rapida, de bate pronto.
 O fato de f'' nao se anular em R implica que f'' eh estritamente positiva ou
 estritamente negativa em R.
 Se f'' for estritamente positiva, f eh convexa.  Alem disto, f' eh
 estritamente crescente em R, de modo que f nao eh constante. Se houver dois
 intervalos fechados [a,b] e [c,d] tais que os segmentos de retas definidos
 pelos pares (a,f(a)) e (b, f(b)) e (c,f(c)) e (d,f(d)) se bisectem, entao os
 intervalos sao encaixados e tem o mesmo ponto medio m =(a+b)/2 = (c+d)/2.
 Mas entao a convexidade de f e o fato de f nao ser constante  acarretam que
 o segmento de reta correspondente ao intervalo externo esteja sempre acima
 do correspondente ao intervalo interno, contrariando a hipotese de que eles
 se bisectem.
 Se f'' for estritamente negativa, entao f eh concava e nao constante,
 cabendo argumentos similares.
 Artur 
 
 
 - Mensagem Original 
 De: [EMAIL PROTECTED]
 Para: Lista OBM [EMAIL PROTECTED]
 Assunto: [obm-l] Cordas no grafico de uma funcao
 Data: 07/10/04 19:26
 
 A funcao f: R - R eh duas vezes diferenciavel e f''(x)  0 para todo x
 real. Prove que duas cordas quaisquer no grafico de f nao se bisectam.
 (uma corda eh um segmento de reta que une dois pontos distintos do grafico
 de f).
 
 []s,
 Claudio.
 

=
Instruções para entrar na lista, sair da lista e usar a lista em
http://www.mat.puc-rio.br/~nicolau/olimp/obm-l.html
=


[obm-l] Soma de números primos

2004-10-13 Por tôpico Marcio M Rocha




 Boatarde a 
todos.

 Gostaria da ajuda 
de vocês com o seguinte problema:

 
“Demonstre que a soma de todos os números primos entre 1 e 2004 é menor que 
667222.”

 Tentei um caminho 
destrutivo, eliminado alguns números que não são primos:

a) 
Da 
seqüência 1, 2, 3, ..., 2004, retirei o 1 e os números pares maiores que 
2.
b) 
Calculei 
a soma S dos termos da seqüência 
restante

S = 2 + 3 + 5 + 7 + 9 +...+ 
2003

obtendo S = 1 004 005.

c) 
Da 
seqüência anterior, eliminei os múltiplos de 3 maiores que 3. Como a soma desses 
múltiplos é 334 665, 
a soma

S1 = 2 + 3 + 5 + 7 + 11 + 13 + 17 + 
... + 2003

vale

S1 = S – 334 665 = 669 
340.

Como a seqüência 2, 3, 5, 7, 11, 
..., 2003 é formada ainda por números compostos, basta que eu retire alguns 
deles, lembrando apenas de não retirar nenhum múltiplo de 3. Retiro, então, 1963 
= 13 x 151 e 155 =5 x 31, e a soma dos números restantes fica igual a 667 
222. Como ainda há números compostos, está claro que a soma dos primos entre 1 e 
2004 deve ser menor que 667 222.

Está tudo OK? 
Alguémpoderiadarum caminho melhor?

Abraços,

Márcio Rocha.

P.S. Embora reconheça que muitos 
participantes da lista não necessitem, gostaria de pedir em meu nome ( e talvez 
no de outros), que as soluções, sempre que possível, viessem acompanhadas das 
"motivações", para que aqueles que lêem não fiquem com a sensação de "coelho 
tirado da cartola". Peço isso porque li um artigo de Miguel de 
Guzmán onde ele diz queEuler, em sua obra,"colocava-se inicialmente na ignorância do 
temae dos métodos queiriaempregar, para começar en condicões 
de igualdade con aquele a quem trata de conduzir pelo caminho,ajudando-o a 
ver as dificuldades que elemesmo encontrou, levando-o,às vezes, 
porcaminhos equivocados queele mesmo havia percorrido antes, a fim 
de que aprenda também dos equívocos". (O artigo completo em espanhol está em www.campus-oei.org/oim/saladelectura.htm, 
sob o título "O papel do matemático en la educación 
matemática"

Se não estiver fora do tema, 
poder-se-ia discutir também estratégias de solução, como as apresentadas no 
Problem Solving Strategies.

Desculpem se escrevi 
demais.


Re: [obm-l] numero primo para (x^2 + 5x+ 23)

2004-10-13 Por tôpico Claudio Buffara
Essa foi uma questao da 3a. fase da obm nivel 3 de 2003.
A resposta eh 17.
Primeiro, verifique que (-2)^2 + 5*(-2) + 23 = 17.
Em seguida, lembre-se de que se p(x) nao eh divisivel por n para n valores
inteiros consecutivos de x, entao p(x) nao eh divisivel por n para nenhum
inteiro x.
O maior primo menor do que 17 eh 13. Assim, tome 13 valores inteiros
consecutivos de x e verifique se, para algum deles, p(x) eh divisivel por
algum primo = 13. Voce vai ver que a resposta eh nao.

[]s,
Claudio.

on 08.10.04 17:29, Osvaldo Mello Sponquiado at [EMAIL PROTECTED] wrote:

 
 Prove que x^2 + 5x + 23 eh sempre impar, qualquer que
 seja x inteiro.
 
 Bom acho que enviei um mail para a lista, mas não
 chegou, aff.
 
 Minha tentativa para encontrar o menor primo que
 dividia x^2 + 5x + 23, para algum x natural foi
 fazer x^2 + 5x + 23=(x+a).(x+a)+1.(x+a)+b; a arbitrario
 natural e b=b(a).
 Adotando a=2 vem b=23-2^2-2=17
 Assim ficamos com x^2 + 5x + 23=(x+2)[(x+2)+1]+17=
 (x+2)(x+3)+17
 
 A parcela (x+2)(x+3) é o produto de dois termos
 consecutivos logo é um número par.
 A segunda parcela é um número ímpar, logo a soma é um
 número impar.
 Assim queremos encontrar o menor p, ímpar, tal que p |
 (x+2)(x+3)+17
 
 Usando congruencia fiz
 Possibilidade 1) (x+2) cong. 17 mod (p)
 e (x+3) cong. 1 mod (p)
 Possibilidade 2) (x+3) cong. 17 mod (p)
 e (x+2) cong. 1 mod (p)
 
 Do caso 1 tiramos:
 x cong. 15 mod (p) e x cong. -2 mod (p)
 dai x=kp+15=lp-2= p=17/(l-k); l e k naturais
 Como p é primo por hipótese e 17 é primo entaum l deve
 ser necessariamente k+1 e logo p=17
 Do 2, tiramos:
 x cong. 14 mod (p) e x cong. -1 mod (p)
 x=ap+14=bp-1=p=15/(b-a)
 Como os divisores positivos de 15 são 1,3,5,15, então
 as possibilidades para p são p=3 e p=5 pois p é primo.
 
 x^2 + 5x + 23=3.7+(x^2+5x+2)=3.6+(x^2+5x+5)
 Se fizermos 3.k; k=5 mna equação acima teremos raízes
 complexas na eq. do segundo grau correspondente por
 exemplo se k=4 fica x^2 + 5x + 23=3.4+(x^2+5x+11), onde
 x^2+5x+11 possui raizes complexas, logo nao se analisa
 estes.
 
 Para k=5 ou k=6 temos que expressao  x^2 + 5x + 23=3k+f
 (x); onde f(x) é a expressao do segundo grau
 correspondente nunca é múltipla de três.
 Procedendo com o mesmo argumento encontramos que x^2 +
 5x + 23 não é múltiplo de cinco.
 
 Será que desta forma estaria correto ?
 Ateh mais.
 
 
 Atenciosamente,
 
 Osvaldo Mello Sponquiado
 Engenharia Elétrica, 2ºano
 UNESP - Ilha Solteira
 
 


=
Instruções para entrar na lista, sair da lista e usar a lista em
http://www.mat.puc-rio.br/~nicolau/olimp/obm-l.html
=


Re: RES: [obm-l] Inversa de uma Matriz

2004-10-13 Por tôpico Claudio Buffara
Oi, Artur:

Tudo bem, mas eu estava tentando provar isso a partir de conceitos mais
basicos, tais como sistemas lineares e matrizes elementares.
O fato de que A eh invertivel se e somente se det(A)  0 eh muito avancado,
mas obviamnete estah correto.

[]s,
Claudio.

on 08.10.04 16:12, Artur Costa Steiner at [EMAIL PROTECTED] wrote:

 O que eh trivial depende da experiencia de cada um
 Mas como AB = I, temos que det(AB) = det(A) * det(B) = det(I) = 10, de
 modo que det(A)0 e det(B)0.  Logo, A e B sao invertiveis. Como a inversa
 de uma matriz nao singular eh unica e AB=I, temos que B = A^(-1), o que
 implica que BA = I.
 Artur
 
 
 - Mensagem Original 
 De: [EMAIL PROTECTED]
 Para: [EMAIL PROTECTED] [EMAIL PROTECTED]
 Assunto: RES: [obm-l] Inversa de uma Matriz
 Data: 08/10/04 11:56
 
 O problema a seguir eh trivial?
 
 Sejam A e B matrizes quadradas tais que AB = I. Prove que BA = I.
 (I = matriz identidade)
 
 

=
Instruções para entrar na lista, sair da lista e usar a lista em
http://www.mat.puc-rio.br/~nicolau/olimp/obm-l.html
=


Re: [obm-l] Inversa de uma Matriz

2004-10-13 Por tôpico Claudio Buffara
on 08.10.04 15:54, Nicolau C. Saldanha at [EMAIL PROTECTED] wrote:

 On Fri, Oct 08, 2004 at 11:05:22AM -0200, Claudio Buffara wrote:
 O problema a seguir eh trivial?
 
 Sejam A e B matrizes quadradas tais que AB = I. Prove que BA = I.
 (I = matriz identidade)
 
 Problema adicional:
 Se A for mxn, B nxm com m  n e AB = I (identidade mxm), o que poderemos
 dizer sobre BA?
 
 Começando pelo segundo problema, podemos dizer que (BA)^2 = B(AB)A = BA
 donde BA é uma projeção de posto m, ou seja, uma projeção de R^n sobre
 um subespaço de dimensão m.
 
 Quanto ao primeiro, eu diria que ele *não* é trivial. Encarando A e B
 como transformações lineares, é bem claro que A é sobre e B é injetora.
 O que fica faltando é provar o seguinte lema:
 
 Seja T uma transformação linear de um espaço vetorial de dimensão finita V
 nele mesmo. Então as seguintes condições são equivalentes:
 
 (a) T é injetora;
 (b) T é sobrejetora;
 (c) T é inversível.
 
 Este é uma espécie de versão linear do princípio das casas de pombos
 e requer demonstração. A demonstração pode ser encontrada em qualquer
 livro de álgebra linear, claro, mas não é de todo trivial. Note que todas
 as seguintes hipóteses são necessárias:
 
 Dimensão finita: o lema é falso em espaços vetoriais de dimensão infinita.
 Espaço vetorial: o lema é falso para módulos sobre quase qualquer anel.
 
 A necessidade destas duas hipóteses torna a meu ver o princípio das
 casas de pombos lineares algo não trivial.
 
 []s, N.
 
 
Oi, Nicolau:

Obrigado pela resposta. Voce iluminou um novo angulo do problema.

Para o primeiro problema, eu havia pensado em usar um resultado que diz
respeito as condicoes minimas necessarias para um semi-grupo ser um grupo.
Acho que o Domingos mencionou algo a respeito. Em linguagem de matrizes
seria o seguinte:

Seja M um conjunto de matrizes quadradas nxn (n arbitrario), fechado em
relacao ao produto usual de matrizes (que sabemos ser associativo) e com as
seguintes propriedades:
1) Existe I em M tal que A*I = A, para toda A em M;
2) Para cada A em M, existe B em M tal que A*B = I.
Entao, para cada A em M vale I*A = A e dada B tal que A*B = I, tem-se B*A =
I.

Tomemos A em M. Seja B tal que A*B = I.
Como B estah em M, vai existir C em M tal que B*C = I.
Entao, A = A*I = A*(B*C) = (A*B)*C = I*C.
Logo, B*A = B*(I*C) = (B*I)*C = B*C = I.
Alem disso, I*A = (A*B)*A = A*(B*A) = A*I = A.


[]s,
Claudio.


=
Instruções para entrar na lista, sair da lista e usar a lista em
http://www.mat.puc-rio.br/~nicolau/olimp/obm-l.html
=


RES: [obm-l] Mais um problema legal

2004-10-13 Por tôpico agatavares
Na realidade a quantidade de somas que têm o zero em uma das parcelas é
1002, o que nos dá
[2004!/(2002! * 2! * 3!)] - 1002.

Acho que agora está correto.

-Mensagem original-
De: [EMAIL PROTECTED] [mailto:[EMAIL PROTECTED]
Enviada em: terça-feira, 12 de outubro de 2004 21:21
Para: [EMAIL PROTECTED]
Assunto: RES: [obm-l] Mais um problema legal


Penso que uma boa seqüência de resolução desse problema seja [2004!/(2002! *
2! * 3!)] - 2003, pois para obtermos 2002 como a soma de três inteiros
positivos, podemos ter

|...||| + |||...|| + |||...|| = 2002
 500600 902

como também

|||...|| + ||...|| + |||...||| = 2002
   200 1000 802

Portanto teríamos todas as permutações possíveis dos 2004 símbolos
(tracinhos e sinais de +), dividido pelas permutações dos tracinhos e também
dos sinais. A divisão pela permutação de 3 é porque cada soma, em função da
ordem não importar, tem 3! repetições. A diminuição de 2003 possibilidades é
nececssária para que se retirem as soluções que têm o zero em uma das
parcelas.

Sou um novo integrante do grupo e pela primeira vez tento enviar alguma
resposta. Espero que consiga. Caso haja equívocos na minha solução, espero
respostas.

Um abraço a todos.

Agamenon.


-Mensagem original-
De: [EMAIL PROTECTED] [mailto:[EMAIL PROTECTED]
nome de benedito
Enviada em: terça-feira, 12 de outubro de 2004 18:08
Para: [EMAIL PROTECTED]
Assunto: [obm-l] Mais um problema legal





 Segue mais um problema interessante (Agora com o problema. Desculpem a
falha).
 Benedito Freire

PROBLEMA

Sem levar em consideração a ordem, de quantas maneiras podemos expressar
2002  como soma de  3  inteiros positivos?

(Atenção: 1000 + 1000 + 3 = 2002   e  1000 + 2 + 1000 = 2002  não são
consideradas maneiras distintas de expressar  2002  como soma de inteiros
positivos)




--
Esta mensagem foi verificada pelo sistema de anti-virus e
 acredita-se estar livre de perigo.

=
Instruções para entrar na lista, sair da lista e usar a lista em
http://www.mat.puc-rio.br/~nicolau/olimp/obm-l.html
=


=
Instruções para entrar na lista, sair da lista e usar a lista em
http://www.mat.puc-rio.br/~nicolau/olimp/obm-l.html
=


[obm-l] Duvida na desigualdade triangular

2004-10-13 Por tôpico Fabio Niski
Primeiramente, obrigado Paulo pela ajuda na questao de convexidade.
Estou com uma duvida elementar...gostaria que por favor me ajudassem.
Lendo uma prova do fato de que se f tem derivada em um ponto c, entao f 
é continua em c, Bartle argumenta que

Seja eps = 1 e tome d = d(1)  tal que
| [(f(x)-f(c))/(x-c)] - f'(c)|  1, para todo x no dominio de f, 
satisfazendo 0  | x - c |  d.
Da desigualdade triangular, nos inferimos que para esses valores de x, temos
|f(x) - f(c)|  = |x-c|{|f'(c)| +1}

Eu realmente nao enxergo como ele chegou nessa ultima desigualdade...
Alguem por favor me ajude!
Obrigado

=
Instruções para entrar na lista, sair da lista e usar a lista em
http://www.mat.puc-rio.br/~nicolau/olimp/obm-l.html
=


Re: [obm-l] Cordas no grafico de uma funcao

2004-10-13 Por tôpico Artur Costa Steiner
Eu achei uma solucao que usou sucessivamente o teorema do valor medio.
No entanto, uma dica que talvez seja util eh a seguinte: se as duas cordas
se bisectam, entao elas sao diagonais de um paralelogramo... pensando
melhor, no fim voce vai precisar do t.v.m de qualquer jeito.

Bom, aquelas outras conclusoes que eu citei sem demonstrar tambem se
baseiam, a menos para as provas que eu conheco, em sucessivas aplicacoes do
t. do valor medio.
Artur


OPEN Internet e Informática
@ Primeiro provedor do DF com anti-vírus no servidor de e-mails @


=
Instruções para entrar na lista, sair da lista e usar a lista em
http://www.mat.puc-rio.br/~nicolau/olimp/obm-l.html
=


Re: RES: [obm-l] Inversa de uma Matriz

2004-10-13 Por tôpico Artur Costa Steiner
Oi, Artur:

Tudo bem, mas eu estava tentando provar isso a partir de conceitos mais
basicos, tais como sistemas lineares e matrizes elementares.
O fato de que A eh invertivel se e somente se det(A)  0 eh muito
avancado,
mas obviamnete estah correto.

OK, mas eu tambem nao estava querendo dizer que era trivial para mim...
A prva que vc apresentou na outra mensagem eh ateh bem mais dificil e mais
geral do que a baseada em determinantes. Eu tambem jaj admiti conhecido que
a inversa de uma matriz nao singular eh unica.
Artur


OPEN Internet e Informática
@ Primeiro provedor do DF com anti-vírus no servidor de e-mails @


=
Instruções para entrar na lista, sair da lista e usar a lista em
http://www.mat.puc-rio.br/~nicolau/olimp/obm-l.html
=


Re: [obm-l] Cordas no grafico de uma funcao

2004-10-13 Por tôpico Claudio Buffara
Oi, Artur:

Tem outra coisa que voce escreveu que me deixou em duvida: o fato de f''
nao se anular em R implica que f'' eh estritamente positiva ou estritamente
negativa. Serah que isso nao pressupoe que f'' eh continua? O enunciado diz
apenas que f''(x)  0 para cada x real mas nao que f'' eh continua.

Por exemplo, se f(x) = x*|x|, teremos f''(x) = -2 para x  0 e f''(x) = 2
para x  0. Nesse caso f''(0) nao existe. De qualquer jeito, f''(x) eh  0
onde eh definida mas f'' nao eh estritamente positiva nem estritamente
negativa.

Talvez voce tenha razao porque, se nao me engano, uma funcao derivada nao
pode ter qualquer tipo de descontinuidade, mas eu estou meio sem saco de
procurar os detalhes num livro...

[]s,
Claudio.

on 13.10.04 14:31, Claudio Buffara at [EMAIL PROTECTED] wrote:

 Eu achei uma solucao que usou sucessivamente o teorema do valor medio.
 No entanto, uma dica que talvez seja util eh a seguinte: se as duas cordas
 se bisectam, entao elas sao diagonais de um paralelogramo... pensando
 melhor, no fim voce vai precisar do t.v.m de qualquer jeito.
 
 []s,
 Claudio.
 
 on 08.10.04 17:45, Artur Costa Steiner at [EMAIL PROTECTED] wrote:
 
 Fazendo uma analise rapida, de bate pronto.
 O fato de f'' nao se anular em R implica que f'' eh estritamente positiva ou
 estritamente negativa em R.
 Se f'' for estritamente positiva, f eh convexa.  Alem disto, f' eh
 estritamente crescente em R, de modo que f nao eh constante. Se houver dois
 intervalos fechados [a,b] e [c,d] tais que os segmentos de retas definidos
 pelos pares (a,f(a)) e (b, f(b)) e (c,f(c)) e (d,f(d)) se bisectem, entao os
 intervalos sao encaixados e tem o mesmo ponto medio m =(a+b)/2 = (c+d)/2.
 Mas entao a convexidade de f e o fato de f nao ser constante  acarretam que
 o segmento de reta correspondente ao intervalo externo esteja sempre acima
 do correspondente ao intervalo interno, contrariando a hipotese de que eles
 se bisectem.
 Se f'' for estritamente negativa, entao f eh concava e nao constante,
 cabendo argumentos similares.
 Artur 
 
 
 - Mensagem Original 
 De: [EMAIL PROTECTED]
 Para: Lista OBM [EMAIL PROTECTED]
 Assunto: [obm-l] Cordas no grafico de uma funcao
 Data: 07/10/04 19:26
 
 A funcao f: R - R eh duas vezes diferenciavel e f''(x)  0 para todo x
 real. Prove que duas cordas quaisquer no grafico de f nao se bisectam.
 (uma corda eh um segmento de reta que une dois pontos distintos do grafico
 de f).
 
 []s,
 Claudio.
 
 

=
Instruções para entrar na lista, sair da lista e usar a lista em
http://www.mat.puc-rio.br/~nicolau/olimp/obm-l.html
=


[obm-l] Variancia Zero

2004-10-13 Por tôpico Claudio Buffara

 Quoting Murilo Neves [EMAIL PROTECTED]:
 
 Olá
 
 Continuo aqui estudando e surgiram mais duas dúvidas. Se alguém puder me
 ajudar... Vamos lá:
 
 1)Sabendo-se que E(X)=2 e que E(X^2)=4, calcule o menor valor possível para
 P(-10X14).
 Meu gabarito dá que a resposta é aproximadamente 0.9. Tentei usar Chebyshev,
 
 mas como Var(X)=E^2(X) - E(X^2)=0, chegaria a resposta 1. Tentei usar
 Markov, mas também não consegui.
 
Uma duvida: se X eh uma var. aleatoria com variancia = 0 entao X eh
necessariamente constante?

[]s,
Claudio.


=
Instruções para entrar na lista, sair da lista e usar a lista em
http://www.mat.puc-rio.br/~nicolau/olimp/obm-l.html
=


Re: RES: [obm-l] Inversa de uma Matriz

2004-10-13 Por tôpico Claudio Buffara
on 13.10.04 17:24, Artur Costa Steiner at [EMAIL PROTECTED] wrote:

 Oi, Artur:
 
 Tudo bem, mas eu estava tentando provar isso a partir de conceitos mais
 basicos, tais como sistemas lineares e matrizes elementares.
 O fato de que A eh invertivel se e somente se det(A)  0 eh muito
 avancado,
 mas obviamnete estah correto.
 
 OK, mas eu tambem nao estava querendo dizer que era trivial para mim...
 A prva que vc apresentou na outra mensagem eh ateh bem mais dificil e mais
 geral do que a baseada em determinantes. Eu tambem jaj admiti conhecido que
 a inversa de uma matriz nao singular eh unica.
 Artur
 
Mas, dado que a inversa existe, a sua unicidade eh realmente facil de
mostrar.
Se AB = BA = AC = CA = I, entao, B = BI = B(AC) = (BA)C = IC = C.
Alias, isso vale para funcoes em geral e nao apenas matrizes ou
transformacoes lineares.

O problema eh que uma transformacao linear pode ter uma inversa a direita e
nao ser invertivel. Um exemplo eh a transformacao derivada no espaco
vetorial dos polinomios. Alias, o Nicolau mencionou este ponto.
Assim, dimensao finita deve ser essencial. Soh que, na minha demonstracao
baseada em grupos, onde eh que dimensao finita entra?

[]s,
Claudio.

=
Instruções para entrar na lista, sair da lista e usar a lista em
http://www.mat.puc-rio.br/~nicolau/olimp/obm-l.html
=


RE: [obm-l] Mais um problema legal

2004-10-13 Por tôpico João Gilberto Ponciano Pereira
Imaginei assim:

Sejam as 3 parcelas X, Y e Z tais que X = Y= Z. Logo, X=667.

1- supondo X fixo ímpar, temos que Y pode variar de X (inclusive) a (2002 -
1 - x)/2. Em outras palavras, quando X=1 temos (2002 - 1 - 1)/2 = 1000
valores possíveis para Y válidos, logo, 1000 possíveis variações. Colocando
numa tabela para facilitar as contas, temos:

  X  - Variações
  1  -  1000
  3  -   997
  5  -   994
..
667  - 1

2- analogamente, supondo um x fixo par, temos que Y pode variar de X
(inclusive) a (2002 - x)/2. Neste caso, a tabela ficaria com a seguinte
cara:

  X  - Variações
  2  -   999
  4  -   996
  6  -   993
..
666  - 3

Fazendo as contas necessárias, chego num número um pouco diferente: 
334000


SDS
JG

-Original Message-
From: [EMAIL PROTECTED] [mailto:[EMAIL PROTECTED]
Sent: Wednesday, October 13, 2004 3:22 PM
To: [EMAIL PROTECTED]
Subject: RES: [obm-l] Mais um problema legal


Na realidade a quantidade de somas que têm o zero em uma das parcelas é
1002, o que nos dá
[2004!/(2002! * 2! * 3!)] - 1002.

Acho que agora está correto.

-Mensagem original-
De: [EMAIL PROTECTED] [mailto:[EMAIL PROTECTED]
Enviada em: terça-feira, 12 de outubro de 2004 21:21
Para: [EMAIL PROTECTED]
Assunto: RES: [obm-l] Mais um problema legal


Penso que uma boa seqüência de resolução desse problema seja [2004!/(2002! *
2! * 3!)] - 2003, pois para obtermos 2002 como a soma de três inteiros
positivos, podemos ter

|...||| + |||...|| + |||...|| = 2002
 500600 902

como também

|||...|| + ||...|| + |||...||| = 2002
   200 1000 802

Portanto teríamos todas as permutações possíveis dos 2004 símbolos
(tracinhos e sinais de +), dividido pelas permutações dos tracinhos e também
dos sinais. A divisão pela permutação de 3 é porque cada soma, em função da
ordem não importar, tem 3! repetições. A diminuição de 2003 possibilidades é
nececssária para que se retirem as soluções que têm o zero em uma das
parcelas.

Sou um novo integrante do grupo e pela primeira vez tento enviar alguma
resposta. Espero que consiga. Caso haja equívocos na minha solução, espero
respostas.

Um abraço a todos.

Agamenon.


-Mensagem original-
De: [EMAIL PROTECTED] [mailto:[EMAIL PROTECTED]
nome de benedito
Enviada em: terça-feira, 12 de outubro de 2004 18:08
Para: [EMAIL PROTECTED]
Assunto: [obm-l] Mais um problema legal





 Segue mais um problema interessante (Agora com o problema. Desculpem a
falha).
 Benedito Freire

PROBLEMA

Sem levar em consideração a ordem, de quantas maneiras podemos expressar
2002  como soma de  3  inteiros positivos?

(Atenção: 1000 + 1000 + 3 = 2002   e  1000 + 2 + 1000 = 2002  não são
consideradas maneiras distintas de expressar  2002  como soma de inteiros
positivos)




--
Esta mensagem foi verificada pelo sistema de anti-virus e
 acredita-se estar livre de perigo.

=
Instruções para entrar na lista, sair da lista e usar a lista em
http://www.mat.puc-rio.br/~nicolau/olimp/obm-l.html
=


=
Instruções para entrar na lista, sair da lista e usar a lista em
http://www.mat.puc-rio.br/~nicolau/olimp/obm-l.html
=

=
Instruções para entrar na lista, sair da lista e usar a lista em
http://www.mat.puc-rio.br/~nicolau/olimp/obm-l.html
=


Re: [obm-l] Mais um problema legal

2004-10-13 Por tôpico Claudio Buffara
on 12.10.04 19:07, benedito at [EMAIL PROTECTED] wrote:

 
 Segue mais um problema interessante (Agora com o problema. Desculpem a
 falha).
 Benedito Freire
 
 PROBLEMA
 
 Sem levar em consideração a ordem, de quantas maneiras podemos expressar
 2002  como soma de  3  inteiros positivos?
 
 (Atenção: 1000 + 1000 + 3 = 2002   e  1000 + 2 + 1000 = 2002  não são
 consideradas maneiras distintas de expressar  2002  como soma de inteiros
 positivos)
 
 
 
Uma ideia eh separar os dois casos:
A + A + B = 2002 com A  B
e 
A + B + C = 2002 com A  B  C.

O primeiro caso eh facil. Obviamente B terah de ser par e as solucoes serao:
A = 1000, B = 2
A = 999, B = 4
...
A = 1, B = 2000
Ou seja, teremos 1000 particoes distintas de 2002 nas quais duas parcelas
sao iguais.

No segundo caso, calculemos inicialmente o numero de solucoes inteiras e
positivas da equacao A + B + C = 2002.
Como todo mundo sabe, esse numero eh Binom(2001,2) = 2.001.000.

Dentre estas, o numero de solucoes onde A = B, A = C ou B = C eh igual a
3*1000 = 3000.

Subtraindo essas 3000 solucoes do total achado acima, obtemos 1.998.000
solucoes em que A, B e C sao mutuamente distintos.

Como ordem nao eh importante, precisamos dividir este numero por 3!, obtendo
333.000 particoes de 2002 em tres parcelas distintas.

Juntamente com as 1000 particoes com duas parcelas iguais, achamos um total
de 334.000 particoes de 2002 em tres parcelas.


[]s,
Claudio.


=
Instruções para entrar na lista, sair da lista e usar a lista em
http://www.mat.puc-rio.br/~nicolau/olimp/obm-l.html
=


Re: [obm-l] Duvida na desigualdade triangular

2004-10-13 Por tôpico Claudio Buffara
on 13.10.04 16:40, Fabio Niski at [EMAIL PROTECTED] wrote:

 Primeiramente, obrigado Paulo pela ajuda na questao de convexidade.
 Estou com uma duvida elementar...gostaria que por favor me ajudassem.
 Lendo uma prova do fato de que se f tem derivada em um ponto c, entao f
 é continua em c, Bartle argumenta que
 
 Seja eps = 1 e tome d = d(1)  tal que
 | [(f(x)-f(c))/(x-c)] - f'(c)|  1, para todo x no dominio de f,
 satisfazendo 0  | x - c |  d.
 Da desigualdade triangular, nos inferimos que para esses valores de x, temos
 |f(x) - f(c)|  = |x-c|{|f'(c)| +1}
 
 Eu realmente nao enxergo como ele chegou nessa ultima desigualdade...
 
 Alguem por favor me ajude!
 
 Obrigado
 
Se A e B sao reais (ou complexos), entao |A - B| = ||A| - |B||  (*)

Pra ver isso, aplique a desigualdade triangular:
|A| = |B + (A - B)| = |B| + |A - B| == |A| - |B| = |A - B|

Permutando A e B, voce obtem |B| - |A| = |B - A| = |A - B|.

Junatando as duas, voce obtem a desigualdade (*) acima.


[]s,
Claudio.


=
Instruções para entrar na lista, sair da lista e usar a lista em
http://www.mat.puc-rio.br/~nicolau/olimp/obm-l.html
=


Re: [obm-l] elementos de ordem 2 em grupos abelianos

2004-10-13 Por tôpico kleinad
Que tal (Z_2)^n = espaco vetorial das n-uplas ordenadas cujas componentes
sao elementos de Z_2, ou seja, 0 ou 1, com a operacao de soma componente a
componente e tal que 0+0 = 1+1 = 0 e 0+1 = 1+0 = 1?

Ok!, embora eu não chamaria isso a rigor de espaço vetorial.

A hipotese de G ser abeliano e o teorema de Cauchy tambem sao
desnecessarios. Todo grupo finito G em que os elementos distintos da
identidade tem ordem 2 eh necessariamente abeliano e tem ordem 2^n para
algum n.

Concordo totalmente com vc, e aliás a primeira parte é trivial; apenas
estava encurtando o trabalho. Como essa minha pergunta se originou de certo
modo de um problema que constava logo após a introdução do teorema de
Cauchy, no livro que uso, preferi não desperdiçar a oportunidade de usá-lo...

[]s,
Daniel

=
Instruções para entrar na lista, sair da lista e usar a lista em
http://www.mat.puc-rio.br/~nicolau/olimp/obm-l.html
=


RE: [obm-l] Duvida na desigualdade triangular

2004-10-13 Por tôpico Paulo Santa Rita
Oi Niski e demais colegas
desta lista ... OBM-L,
Nao ha o que agradecer. Gostaria de ter tempo para poder participar mais, 
conforme eu fazia em tempos idos. Nao sei se entendi o que voce quer abaixo, 
mas pode suceder que seja tao obvio que voce nao esta vendo. Imagino que ele 
usa : |a| - |b| = |a-b| e |a/b| = |a|/|b|. Senao, vejamos :

| [(f(x)-f(c))/(x-c)] | - | f'(c)| = | [(f(x)-f(c))/(x-c)] - f'(c)|  
propriedade dos modulos
Como  | [(f(x)-f(c))/(x-c)] - f'(c)|  1  = | [(f(x)-f(c))/(x-c)] | - | 
f'(c)|  1. Daqui :
| [(f(x)-f(c))/(x-c)] |  | f'(c)| + 1 = | f(x)-f(c)| / |x-c|  | f'(c)| + 
1 Ou seja ;
| f(x) - f(c) |  |x-c|*{|f'(c)| + 1} e evidentemente lim f(x) = f(c) = 
f(x) e continua em c.

Note que os passos acima sao altamente truculentos e desnecessarios, pois, 
se a derivada existe e claramente x-c - 0 ( quando x - c ) entao lim 
{(x-c)*[f(x) - f(c)/x-c]}=0 = lim f(x) = f(c) logo f(x) e continua em c.

Um Abracao
Paulo Santa Rita
4,1838,131004

From: Fabio Niski [EMAIL PROTECTED]
Reply-To: [EMAIL PROTECTED]
To: [EMAIL PROTECTED]
Subject: [obm-l] Duvida na desigualdade triangular
Date: Wed, 13 Oct 2004 15:40:13 -0300
Primeiramente, obrigado Paulo pela ajuda na questao de convexidade.
Estou com uma duvida elementar...gostaria que por favor me ajudassem.
Lendo uma prova do fato de que se f tem derivada em um ponto c, entao f é 
continua em c, Bartle argumenta que

Seja eps = 1 e tome d = d(1)  tal que
| [(f(x)-f(c))/(x-c)] - f'(c)|  1, para todo x no dominio de f, 
satisfazendo 0  | x - c |  d.
Da desigualdade triangular, nos inferimos que para esses valores de x, 
temos
|f(x) - f(c)|  = |x-c|{|f'(c)| +1}

Eu realmente nao enxergo como ele chegou nessa ultima desigualdade...
Alguem por favor me ajude!
Obrigado

=
Instruções para entrar na lista, sair da lista e usar a lista em
http://www.mat.puc-rio.br/~nicolau/olimp/obm-l.html
=
_
MSN Messenger: converse com os seus amigos online.  
http://messenger.msn.com.br

=
Instruções para entrar na lista, sair da lista e usar a lista em
http://www.mat.puc-rio.br/~nicolau/olimp/obm-l.html
=


Re: [obm-l] Cordas no grafico de uma funcao

2004-10-13 Por tôpico Artur Costa Steiner
Oi Claudio,
O que acontece com relacao a derivadas eh o seguinte.

(1) Derivadas podem ser descontinuas sim, mas jamais apresentam
descontinuidade do tipo salto, aquela caracterizada pela existencia de
limites aa direita e aa esquerda mas em valores distintos. Assim, se f' for
descontinua em a, entao f' tem limite em a, apresentando  entao  aquela
descontinuidade que alguns autores chamam de essencial ou nao removivel.
Uma forma facil de ver isto eh usar a Regra de L'Hospital. Eh por isso que
se f' eh monotona em [a,b], entao f' eh continua em [a,b], visto que funcoes
monotonas so podem apresentar descontinuidades do tipo salto.

(2) Derivadas, ainda que nao sejam continuas em seu dominio, apresentam
sempre a chamada propriedade do valor intermediario. Isto eh, se f' eh
definida em [a, b] e f'(a)  f'(b), entao f' assume em [a,b] todos os
valores entre f'(a) e f'(b). Assim, se k estah entre f'(a) e f'(b), podemos
entao afirmar que existe algum c em (a,b) tal que f'(c) = k. Esta conclusao
eh conhecida por Teorema de Darbaux (um matematico frances) e sua
demonstracao eh surpreendentemente simples. Basta considerar a funcao
g:[a,b] - R dada por g(x) = f(x) - k*x e analisa-la quanto a maximos ou
minimos. Assim, se f' nunca se anula em um intervalo, limitado ou nao, entao
f' tem que ser estritamente positiva ou negativa neste intervalo. Foi nisso
que me baseei para no caso das cordas afirmar que f'' era estritamente
positiva ou negatia. Esqueci de fazer este comentario na ocasiao.  

Abracos
Artur 

- Mensagem Original 
De: [EMAIL PROTECTED]
Para: [EMAIL PROTECTED] [EMAIL PROTECTED]
Assunto: Re: [obm-l] Cordas no grafico de uma funcao
Data: 13/10/04 17:00

Oi, Artur:

Tem outra coisa que voce escreveu que me deixou em duvida: o fato de f''
nao se anular em R implica que f'' eh estritamente positiva ou estritamente
negativa. Serah que isso nao pressupoe que f'' eh continua? O enunciado diz
apenas que f''(x)  0 para cada x real mas nao que f'' eh continua.

Por exemplo, se f(x) = x*|x|, teremos f''(x) = -2 para x  0 e f''(x) = 2
para x  0. Nesse caso f''(0) nao existe. De qualquer jeito, f''(x) eh  0
onde eh definida mas f'' nao eh estritamente positiva nem estritamente
negativa.

Talvez voce tenha razao porque, se nao me engano, uma funcao derivada nao
pode ter qualquer tipo de descontinuidade, mas eu estou meio sem saco de
procurar os detalhes num livro...




OPEN Internet e Informática
@ Primeiro provedor do DF com anti-vírus no servidor de e-mails @


=
Instruções para entrar na lista, sair da lista e usar a lista em
http://www.mat.puc-rio.br/~nicolau/olimp/obm-l.html
=


Re: [obm-l] Duvida na desigualdade triangular

2004-10-13 Por tôpico Fabio Niski
Claudio Buffara wrote:
on 13.10.04 16:40, Fabio Niski at [EMAIL PROTECTED] wrote:
 

Primeiramente, obrigado Paulo pela ajuda na questao de convexidade.
Estou com uma duvida elementar...gostaria que por favor me ajudassem.
Lendo uma prova do fato de que se f tem derivada em um ponto c, entao f
é continua em c, Bartle argumenta que
Seja eps = 1 e tome d = d(1)  tal que
| [(f(x)-f(c))/(x-c)] - f'(c)|  1, para todo x no dominio de f,
satisfazendo 0  | x - c |  d.
Da desigualdade triangular, nos inferimos que para esses valores de x, temos
|f(x) - f(c)|  = |x-c|{|f'(c)| +1}
Eu realmente nao enxergo como ele chegou nessa ultima desigualdade...
Alguem por favor me ajude!
Obrigado
   

Se A e B sao reais (ou complexos), entao |A - B| = ||A| - |B||  (*)
 

É essa desigualdade acaba com o meu problema! Tinha esquecido dela!
Obrigado!
=
Instruções para entrar na lista, sair da lista e usar a lista em
http://www.mat.puc-rio.br/~nicolau/olimp/obm-l.html
=


Re: [obm-l] Duvida na desigualdade triangular

2004-10-13 Por tôpico Artur Costa Steiner
Esta demonstracao do Bartle, baseada na definicao de derivada, eh legal, mas
hah uma outra que me parece mais simples de entender e que aparece em um
outro livro do proprio Bartle (e que parece ser mais comum). Suponhamos que
f seja difererenciavel em a e observemos que, para xa em uma vizinhanca de
a, f(x) - f(a) = (x-a) * ((f(x) - f(a))/(x-a). Quando x-a, x-a -0 e 
((f(x) - f(a))/(x-a) - f'(a). Como esta dua funcoes apresentam limite em a,
o produto tambem apresenta, tendo-se que f(x) - f(a) - 0 * f'(a) = 0, o que
prova a continuidade de f em a. 
Isto vale tambem para funcoes complexas de uma variavel.
Artur 


- Mensagem Original 
De: [EMAIL PROTECTED]
Para: [EMAIL PROTECTED] [EMAIL PROTECTED]
Assunto: Re: [obm-l] Duvida na desigualdade triangular
Data: 13/10/04 18:01

on 13.10.04 16:40, Fabio Niski at [EMAIL PROTECTED] wrote:

 Primeiramente, obrigado Paulo pela ajuda na questao de convexidade.
 Estou com uma duvida elementar...gostaria que por favor me ajudassem.
 Lendo uma prova do fato de que se f tem derivada em um ponto c, entao f
 é continua em c, Bartle argumenta que
 
 Seja eps = 1 e tome d = d(1) tal que
 | [(f(x)-f(c))/(x-c)] - f'(c)|  1, para todo x no dominio de f,
 satisfazendo 0  | x - c |  d.
 Da desigualdade triangular, nos inferimos que para esses valores de x,
temos
 |f(x) - f(c)| = |x-c|{|f'(c)| +1}
 
 Eu realmente nao enxergo como ele chegou nessa ultima desigualdade...
 
 Alguem por favor me ajude!
 
 Obrigado
 
Se A e B sao reais (ou complexos), entao |A - B| = ||A| - |B|| (*)

Pra ver isso, aplique a desigualdade triangular:
|A| = |B + (A - B)| = |B| + |A - B| == |A| - |B| = |A - B|

Permutando A e B, voce obtem |B| - |A| = |B - A| = |A - B|.

Junatando as duas, voce obtem a desigualdade (*) acima.


[]s,
Claudio.


=
Instruções para entrar na lista, sair da lista e usar a lista em
http://www.mat.puc-rio.br/~nicolau/olimp/obm-l.html
=


OPEN Internet e Informática
@ Primeiro provedor do DF com anti-vírus no servidor de e-mails @


=
Instruções para entrar na lista, sair da lista e usar a lista em
http://www.mat.puc-rio.br/~nicolau/olimp/obm-l.html
=


[obm-l] Res: Variância Zero

2004-10-13 Por tôpico Murilo Neves
Olá
Primeiro, vou corrigir a besteira que escrevi: Var(X)=E(X^2)- E^2(X) =0 
(tinha escrito ao contrário)
Agora, uma passagem do livro de probabilidade do Barry James(pg.125):..se 
Var(X)=0 então X é constante , com probabilidade 1 (é constante quase 
certamente)...

_
MSN Hotmail, o maior webmail do Brasil.  http://www.hotmail.com
=
Instruções para entrar na lista, sair da lista e usar a lista em
http://www.mat.puc-rio.br/~nicolau/olimp/obm-l.html
=


[obm-l] continuidade e convergencia uniforme

2004-10-13 Por tôpico eritotutor
 
Boa noite amigos, nao esqueçam dessa por favor...

Seja f: R^2 em R definida por:

f(x,y) = (xy^2)/x^2 + y^4, se (x,y) diferente de (0,0)
   = 0, se  (x,y)=(0,0)
Determine o conjunto de pontos onde f eh continua.

2) Prove que a serie:
somátorio com n variando de 1 a infinito de 
x/n(1+nx^2) converge uniformemente em toda reta real.

Desde jah agradeço.

[]s


 

__
Acabe com aquelas janelinhas que pulam na sua tela.
AntiPop-up UOL - É grátis!
http://antipopup.uol.com.br/




=
Instruções para entrar na lista, sair da lista e usar a 
lista em
http://www.mat.puc-rio.br/~nicolau/olimp/obm-l.html

=

 
__
Acabe com aquelas janelinhas que pulam na sua tela.
AntiPop-up UOL - É grátis!
http://antipopup.uol.com.br/



=
Instruções para entrar na lista, sair da lista e usar a lista em
http://www.mat.puc-rio.br/~nicolau/olimp/obm-l.html
=


[obm-l] Provar uma congruencia

2004-10-13 Por tôpico Demetrio Freitas
Ola,

Gostaria de provar uma congruencia. 

Dado F(n) = n^5 -20*n^4 +40*n^3 +70*n^2 +79*n -50 
Prove que F(n) = 0 (mod 120), se n for primo  7.
(Onde = denota conguente)

Por exemplo:
F(11) = -69240 = -120 * 577
F(19) = 170760 =  120 * 1423
F(97) = 6853927800 = 120 * 57116065
F(563) = 54562015773960 = 120 * 454683464783

Porem:
F(15) = -101240 - nao divisivel por 120
F(129) = 30271636600 - nao divisivel por 120
F(597) = 73303331579800 - nao divisivel por 120


Qual caminho usar?

Obrigado,

Demetrio

OBS:
Naturalmente a condição eh se n primo e não sse (se
e somente se), pois ha muitos n compostos onde F(n) 
= 0 (mod 120)






___ 
Yahoo! Acesso Grátis - Internet rápida e grátis. Instale o discador agora! 
http://br.acesso.yahoo.com/
=
Instruções para entrar na lista, sair da lista e usar a lista em
http://www.mat.puc-rio.br/~nicolau/olimp/obm-l.html
=


Re: [obm-l] Res: Variância Zero

2004-10-13 Por tôpico Claudio Buffara
on 13.10.04 19:55, Murilo Neves at [EMAIL PROTECTED] wrote:

 Olá
 
 Primeiro, vou corrigir a besteira que escrevi: Var(X)=E(X^2)- E^2(X) =0
 (tinha escrito ao contrário)
 Agora, uma passagem do livro de probabilidade do Barry James(pg.125):..se
 Var(X)=0 então X é constante , com probabilidade 1 (é constante quase
 certamente)...
 
Um exercicio que talvez valha a pena eh tentar dar um exemplo de uma
variavel aleatoria nao constante cuja variancia eh zero.

[]s,
Claudio.


=
Instruções para entrar na lista, sair da lista e usar a lista em
http://www.mat.puc-rio.br/~nicolau/olimp/obm-l.html
=


[obm-l] UM PROBLEMA DE CONTAGEM!

2004-10-13 Por tôpico jorgeluis
Ok! Felipe e demais colegas!

Considere sobre cada lado de um triângulo equilátero n-1 pontos que, juntamente
com os vértices, dividem cada lado em n segmentos de mesmo comprimento.
Ligando-se todos esses pontos, dois a dois, por meio de segmentos paralelos aos
lados, muitos triângulos equiláteros, de vários tamanhos, são obtidos. Qual é,
em função de n, o número total de tais triângulos?

NOTA: Este trabalhoso problema foi proposto em uma Olimpíada de Matemática da
Unicamp. Sem sombra de dúvidas, deve ter consumido todo o tempo da prova. Será
que existe outra saída mais prática através de figuras evitando assim a enorme
calculeira já que Uma figura vale mil palavras.

A propósito, qual a maior medida: 99^100 ou 100^99?   Abraços!



__
WebMail UNIFOR - http://www.unifor.br.
=
Instruções para entrar na lista, sair da lista e usar a lista em
http://www.mat.puc-rio.br/~nicolau/olimp/obm-l.html
=


Re: [obm-l] Provar uma congruencia

2004-10-13 Por tôpico Claudio Buffara
on 13.10.04 20:19, Demetrio Freitas at [EMAIL PROTECTED]
wrote:

 Ola,
 
 Gostaria de provar uma congruencia.
 
 Dado F(n) = n^5 -20*n^4 +40*n^3 +70*n^2 +79*n -50
 Prove que F(n) = 0 (mod 120), se n for primo  7.
 (Onde = denota conguente)
 
 Por exemplo:
 F(11) = -69240 = -120 * 577
 F(19) = 170760 =  120 * 1423
 F(97) = 6853927800 = 120 * 57116065
 F(563) = 54562015773960 = 120 * 454683464783
 
 Porem:
 F(15) = -101240 - nao divisivel por 120
 F(129) = 30271636600 - nao divisivel por 120
 F(597) = 73303331579800 - nao divisivel por 120
 
 
 Qual caminho usar?
 
 Obrigado,
 
 Demetrio
 
 OBS:
 Naturalmente a condição eh se n primo e não sse (se
 e somente se), pois ha muitos n compostos onde F(n)
 = 0 (mod 120)
 

A primeira coisa eh decompor 120 em fatores primos:
120 = 2^3*3*5.

Agora, basta provar que F(n) == 0 mod 3, 5 e 8 para n primo  7.

Para cada um dos 3 modulos, a ideia eh reduzir F(n) usando propriedades das
congruencias e o pequeno teorema de Fermat.

Mod 3:
F(n) = n^5 -20*n^4 +40*n^3 +70*n^2 +79*n - 50 ==
F(n) == n + n^2 + n + n^2 + n + 1 ==
F(n) == 2*n^2 + 1

Se n for multiplo de 3, entao F(n) == 1 (mod 3).
No entanto, todos os primos  7 sao impares e nao multiplos de 3, de forma
que os seus quadrados sao todos == 1 (mod 3).
Logo, para n primo  7, 2*n^2 + 1 == 2*1 + 1 == 0 (mod 3)

***

Mod 5:
F(n) = n^5 -20*n^4 +40*n^3 +70*n^2 +79*n - 50 ==
F(n) = n - 0 + 0 + 0 - n + 0 ==
F(n) == 0

Ou seja, F(n) eh multiplo de 5 para qualquer inteiro n.

***

Mod 8:
F(n) = n^5 -20*n^4 +40*n^3 +70*n^2 +79*n - 50 ==
F(n) == n^5 + 4*n^4 + 0 - 2*n^2 - n - 2 ==
F(n) == n*n^4 + 4*n^4 - 2*n^2 - n - 2

O quadrado de cada impar eh == 1 (mod 8). Assim, para n impar, teremos:
F(n) == n*1 + 4*1 - 2*1 - n - 2 == 0 (mod 8).

Ou seja, para n impar e nao multiplo de 3, F(n) == 0 (mod 3*5*8).
Em particular, para cada primo n  7, F(n) eh divisivel por n.

Repare que, no seu exemplo acima, 15, 129 e 597 sao todos multiplos de 3.

[]s,
Claudio.


=
Instruções para entrar na lista, sair da lista e usar a lista em
http://www.mat.puc-rio.br/~nicolau/olimp/obm-l.html
=


[obm-l] O PROBLEMA DE NAPOLEÃO!

2004-10-13 Por tôpico jorgeluis
Turma! Sabe-se que Napoleão Bonaparte gostava muito de Matemática e tinha
especial interesse em geometria. Entretanto, é pouco provável que tenha sido
ele mesmo o autor deste problema. O que se sabe com certeza é que Napoleão
manteve relações muito próximas com grandes matemáticos de sua época como
Monge, Lagrange e Laplace, e chegou a nomear este último como seu ministro de
engenharia.

Ao anexar três triângulos eqüiláteros aos lados de um triângulo qualquer, os
centros desses triângulos são vértices de um triângulo eqüilátero.

Afinal! quantos triângulos são formados ao traçarmos as diagonais de um
pentágono regular?

Abraços!



__
WebMail UNIFOR - http://www.unifor.br.
=
Instruções para entrar na lista, sair da lista e usar a lista em
http://www.mat.puc-rio.br/~nicolau/olimp/obm-l.html
=


[obm-l] POSCOMP

2004-10-13 Por tôpico Fabricio Benevides
Caros membros da lista,

hoje à tarde foi divulgado o resultado do POSCOM. Para quem não sabe, este é um exame, nos moldes do GRE, promovido pela Soc. Bras. de Computação. As questões da prova são em geral fáceis, mas o tempo para resolve-las é curto. Ela é dividida em 3 partes: matemática (pura), ciências da comp e tecnologia.

No site: www.sbc.org.br/poscomp/ vocês podem encontrar provas de exemplo. (Mas não as provas reais, que costumam ser mais difíceis). Se alguem aí fez a prova também já pode consultar o resultado no site.

A nota obtida neste exame é levada em consideração na seleção dos mestrados em computação da maioria da universidades brasileiras. (Por tanto, o público alvo são os que estão nos dois ultimos anos de graduação em comp).

A média nacional foi de 24,4 questões de 70 (muito baixa, como de costume). Gostaria de saber, daqueles que fazem faculdade, se vocês tem ideia de como o pessoal da sua faculdade se saiu na prova.


P.S.: Sei que esta, é uma lista de matemática, mas na minha opnião comp e mat tem muita coisa em comum e sei que muitos da lista cursam computção ou têm interesses na área.
		Yahoo! Acesso Grátis - Internet rápida e grátis. Instale o discador agora!

[obm-l] UMA PEQUENA PÉROLA!

2004-10-13 Por tôpico jorgeluis
Dado um quadrado, quantos triângulos equiláteros existem, que possuem os três
vértices sobre os lados do quadrado? Justifique.

Vocês sabiam...que a célebre igualdade e^i.pi + 1 = 0, que contém os 5 números
mais significativos da Matemática, mereceu de vários matemáticos frases
apaixonadas..esta mais surpreendente jóia..., a mais notável fórmula da
Matemática. (R. Feynman, prêmio Nobel de Física)

A propósito, como encontrar o centro de gravidade de uma placa metálica
homogênea em forma de L , sem usar nada mais que uma régua, não graduada? E
para obter uma cruz grega a partir de uma fôlha de papel com apenas um corte
reto, quantas vezes preciso dobrar a fôlha?

Divirtam-se!



__
WebMail UNIFOR - http://www.unifor.br.
=
Instruções para entrar na lista, sair da lista e usar a lista em
http://www.mat.puc-rio.br/~nicolau/olimp/obm-l.html
=


[obm-l] O PARADOXO DE CANTOR!

2004-10-13 Por tôpico jorgeluis
Turma! Vamos descrever um dos primeiros paradoxos da teoria dos conjuntos,
surgido com o próprio Cantor. Aceitando a definição de conjunto dada por
Cantor, podemos conceber o conjunto U de todos os conjuntos. Esse conjunto U
seria, por assim dizer, o conjunto universal; portanto, teria potência máxima,
já que reuniria todos os conjuntos passíveis de consideração. Em particular,
ele teria de ser um elemento de si mesmo, o que já é, em si, um pouco estranho.
Pior que isso é que, ao considerarmos o conjunto P(U), somos levados, pelo
próprio teorema de Cantor, a concluir que P(U)U. Ora, isso contradiz a
hipótese inicial de que existe um conjunto universal U, ou o conjunto de todos
os conjuntos.

Afinal! Por que surgem paradoxos? Nada há de errado com o raciocínio que acaba
nos levando tanto ao paradoxo de Cantor como ao de Russel. Se não há nada de
errado, como então fomos chegar a esses paradoxos? Abraços!



__
WebMail UNIFOR - http://www.unifor.br.
=
Instruções para entrar na lista, sair da lista e usar a lista em
http://www.mat.puc-rio.br/~nicolau/olimp/obm-l.html
=


[obm-l] A PROVA DA IRRACIONALIDADE!

2004-10-13 Por tôpico jorgeluis
A prova da irracionalidade da raiz de 2 é simples, elegante e muito instrutiva
pois utiliza o chamado método de redução ao absurdo. Este tipo de demonstração
também costuma ser denominado prova por contradição e, em sua essência,
constitui-se em supor o contrário daquilo que se deseja demonstrar e concluir
que tal negativa leva a algum absurdo ou contradição. Se o contrário de algo é
um absurdo, logo aquele algo é verdadeiro: esta é a lógica do método. (Alguns
importantes teoremas dos Elementos foram demonstrados por Euclides utilizando a
idéia de redução ao absurdo, o que comprova que ela já era conhecida desde os
primórdios da Matemática dedutiva). Suponhamos, então, que 2^1/2 seja um número
de forma a/b, com a e b inteiros, e que esta fração esteja reduzida a sua forma
mais simples, ou seja, que a e b não tenham fatores comuns (esta simplificação
é sempre possível, como sabemos da Aritmética). Assim a/b = 2^1/2 e a^2/b^2 = 2
então, a^2 = 2b^2 significa que a^2 é um número par, de onde se conclui que a
também é par, digamos 2p. Desta forma (2p)^2 = 2b^2 então, 2p^2 = b^2. Esta
igualdade indica que b^2 é par, ou seja, que b é par. Logo a e b são pares mas
isto é uma contradição com nossa hipótese inicial de que a e b não têm fatores
comuns. Como a única causa possível de termos chegado a este absurdo foi a
suposição de 2^1/2 = a/b, fica provado que 2^1/2 não pode ser o quociente entre
dois números inteiros. Após a raiz de 2, foram descobertos infinitos outros
números irracionais e as coisas ficaram assim até que, no século XVII,
principalmente devido às técnicas do Cálculo Diferencial, funções e números
passaram a poder ser expressos através das séries infinitas.

A propósito, como poderá explicar aos alunos porque 10^(1/3) é irracional, sem
saber o seu valor certo?

Abraços!



__
WebMail UNIFOR - http://www.unifor.br.
=
Instruções para entrar na lista, sair da lista e usar a lista em
http://www.mat.puc-rio.br/~nicolau/olimp/obm-l.html
=


RE: [obm-l] A PROVA DA IRRACIONALIDADE!

2004-10-13 Por tôpico Edward Elric
Totalmente analogo a demonstraçao de 2^1/2:
Suponha racional, assim 10^1/3 é da forma p/q, e podemos considerar 
mdc(p,q)=1 sem perdas.
Assim p^3/q^3=10 - p^3=2*5*q^3, logo p^3 é par, logo p é da forma 2*k, 
Entao:
8*k^3=2*5*q^3 - 5*q^3=2*2*k^3, logo 5*q^3 é par, logo q é da forma 2*j, 
absurdo pois mdc(p,q)=1.


From: [EMAIL PROTECTED]
Reply-To: [EMAIL PROTECTED]
To: [EMAIL PROTECTED]
Subject: [obm-l] A PROVA DA IRRACIONALIDADE!
Date: Wed, 13 Oct 2004 21:35:24 -0300
A prova da irracionalidade da raiz de 2 é simples, elegante e muito 
instrutiva
pois utiliza o chamado método de redução ao absurdo. Este tipo de 
demonstração
também costuma ser denominado prova por contradição e, em sua essência,
constitui-se em supor o contrário daquilo que se deseja demonstrar e 
concluir
que tal negativa leva a algum absurdo ou contradição. Se o contrário de 
algo é
um absurdo, logo aquele algo é verdadeiro: esta é a lógica do método. 
(Alguns
importantes teoremas dos Elementos foram demonstrados por Euclides 
utilizando a
idéia de redução ao absurdo, o que comprova que ela já era conhecida desde 
os
primórdios da Matemática dedutiva). Suponhamos, então, que 2^1/2 seja um 
número
de forma a/b, com a e b inteiros, e que esta fração esteja reduzida a sua 
forma
mais simples, ou seja, que a e b não tenham fatores comuns (esta 
simplificação
é sempre possível, como sabemos da Aritmética). Assim a/b = 2^1/2 e a^2/b^2 
= 2
então, a^2 = 2b^2 significa que a^2 é um número par, de onde se conclui que 
a
também é par, digamos 2p. Desta forma (2p)^2 = 2b^2 então, 2p^2 = b^2. Esta
igualdade indica que b^2 é par, ou seja, que b é par. Logo a e b são pares 
mas
isto é uma contradição com nossa hipótese inicial de que a e b não têm 
fatores
comuns. Como a única causa possível de termos chegado a este absurdo foi a
suposição de 2^1/2 = a/b, fica provado que 2^1/2 não pode ser o quociente 
entre
dois números inteiros. Após a raiz de 2, foram descobertos infinitos outros
números irracionais e as coisas ficaram assim até que, no século XVII,
principalmente devido às técnicas do Cálculo Diferencial, funções e números
passaram a poder ser expressos através das séries infinitas.

A propósito, como poderá explicar aos alunos porque 10^(1/3) é irracional, 
sem
saber o seu valor certo?

Abraços!

__
WebMail UNIFOR - http://www.unifor.br.
=
Instruções para entrar na lista, sair da lista e usar a lista em
http://www.mat.puc-rio.br/~nicolau/olimp/obm-l.html
=
_
MSN Messenger: converse com os seus amigos online.  
http://messenger.msn.com.br

=
Instruções para entrar na lista, sair da lista e usar a lista em
http://www.mat.puc-rio.br/~nicolau/olimp/obm-l.html
=


Re: [obm-l] continuidade e convergencia uniforme

2004-10-13 Por tôpico Artur Costa Steiner
Eu ja enviei uma mensagem sobre isto Artur
--- [EMAIL PROTECTED] [EMAIL PROTECTED]
wrote:
  
 Boa noite amigos, nao esqueçam dessa por favor...
 
 Seja f: R^2 em R definida por:
 
 f(x,y) = (xy^2)/x^2 + y^4, se (x,y) diferente de
(0,0)
= 0, se  (x,y)=(0,0)
 Determine o conjunto de pontos onde f eh continua.
 
 2) Prove que a serie:
 somátorio com n variando de 1 a infinito de 
 x/n(1+nx^2) converge uniformemente em toda reta
real.
 
 Desde jah agradeço.
 
 []s
 
 
  


 __
 Acabe com aquelas janelinhas que pulam na sua tela.
 AntiPop-up UOL - É grátis!
 http://antipopup.uol.com.br/
 
 
 


 =
 Instruções para entrar na lista, sair da lista e
usar a 
 lista em
 http://www.mat.puc-rio.br/~nicolau/olimp/obm-l.html


 =
 
  

__
 Acabe com aquelas janelinhas que pulam na sua tela.
 AntiPop-up UOL - É grátis!
 http://antipopup.uol.com.br/
 
 
 

=
 Instruções para entrar na lista, sair da lista e
usar a lista em
 http://www.mat.puc-rio.br/~nicolau/olimp/obm-l.html

=




___
Do you Yahoo!?
Declare Yourself - Register online to vote today!
http://vote.yahoo.com
=
Instruções para entrar na lista, sair da lista e usar a lista em
http://www.mat.puc-rio.br/~nicolau/olimp/obm-l.html
=


Re: [obm-l] O PARADOXO DE CANTOR!

2004-10-13 Por tôpico Ricardo Bittencourt
[EMAIL PROTECTED] wrote:
Afinal! Por que surgem paradoxos? Nada há de errado com o raciocínio que acaba
nos levando tanto ao paradoxo de Cantor como ao de Russel. Se não há nada de
errado, como então fomos chegar a esses paradoxos? Abraços!
Se você usou um raciocínio perfeito, que segue todas
regras do cálculo proposicional, e chegou em um paradoxo, é
porque sua premissa estava errada (ou seja, seus axiomas eram
internamente inconsistentes).

Ricardo Bittencourt   http://www.mundobizarro.tk
[EMAIL PROTECTED]  kimitatino kitiwa subete CATS ga itadaita
-- União contra o forward - crie suas proprias piadas --
=
Instruções para entrar na lista, sair da lista e usar a lista em
http://www.mat.puc-rio.br/~nicolau/olimp/obm-l.html
=


Re: [obm-l] UM PROBLEMA DE CONTAGEM!

2004-10-13 Por tôpico Claudio Buffara
Uma ideia eh usar que (1+1/n)^n eh uma sequencia monotona crescente que
converge pra e:

100^99/99^100 = (1/99)*(100/99)^99 = (1/99)*(1 + 1/99)^99  e/99  1 ==
100/99  99^100.

[]s,
Claudio.

on 14.10.04 02:02, Ricardo Bittencourt at [EMAIL PROTECTED] wrote:

 [EMAIL PROTECTED] wrote:
 
 A propósito, qual a maior medida: 99^100 ou 100^99?   Abraços!
 
 Deve ter jeito fácil de fazer, eu naturalmente só sei
 o jeito difícil hehe
 
 Considere o binômio de Newton:
 
 (a+b)^n=sum[1,n]{binomial(n,i).a^(n-i).b^i}
 
 Substituindo a=99, n=99, b=1:
 
 100^99=(99+1)^99=
 sum[1,99]{binomial(99,i).99^(99-i).1^99}=
 sum[1,99]{binomial(99,i).99^(99-i)}=
 sum[1,99]{99!/i!/(99-i)! .99^(99-i)}=
 sum[1,99]{prod[1,i]{100-i}/i!.99^(99-i)}
 
 Agora basta ver o conteúdo de cada termo do somatório.
 Cada termo é formado de um número de parcelas. Primeiro eu
 tenho (i) parcelas, que vão de 99 a (99-i+1), ou seja, todos
 elas são menores ou igual a 99. Depois eu tenho mais
 (99-i) parcelas iguais a 99, que obviamente são menores
 ou iguais a 99. Ou seja, no total eu tenho (i+99-i)=99 parcelas
 menores ou iguais a 99. Como isso ainda vai dividido por i!,
 então cada termo do somatório é menor que 99^99 (exceto
 quando i=1, mas isso não afeta o raciocínio). Então:
 
 sum[1,99]{prod[1,i]{100-i}/i!.99^(99-i)}  99.(99^99)
 100^99  99(99^99)
 100^99  99^100
 
 
 Ricardo Bittencourt   http://www.mundobizarro.tk
 [EMAIL PROTECTED]  kimitatino kitiwa subete CATS ga itadaita
 -- União contra o forward - crie suas proprias piadas --
 =
 Instruções para entrar na lista, sair da lista e usar a lista em
 http://www.mat.puc-rio.br/~nicolau/olimp/obm-l.html
 =
 


=
Instruções para entrar na lista, sair da lista e usar a lista em
http://www.mat.puc-rio.br/~nicolau/olimp/obm-l.html
=